Course Content
All Previous Years Krok 2 Papers with Explanations
About Lesson

Question From (1 To 50 )

years, which is associated with difficult childbirth, complicated by massive bleeding, weight loss, brittleness and hair loss, lack of appetite, and depression. Objective examination: uterus and appendages without pathological changes. What is the pathogenesis of the disease? select one:

A. With overproduction of prolactin

b. With reduced production of gonadotropins

C. With reduced progesterone production

D. With overproduction of estrogen

E. With androgen overproduction


The correct answer is: With reduced production of gonadotropins

Explanation

The correct answer to the question is B. With reduced production of gonadotropins.   The patient in this scenario presents with amenorrhea for 3 years, along with a history of difficult childbirth complicated by massive bleeding, weight loss, hair loss, lack of appetite, and depression.

On examination, the uterus and appendages appear normal. These findings are consistent with a diagnosis of hypothalamic amenorrhea, which is a type of secondary amenorrhea that occurs as a result of reduced production of gonadotropins by the pituitary gland.   Gonadotropins, such as follicle-stimulating hormone (FSH) and luteinizing hormone (LH), are essential for the regulation of the menstrual cycle and the production of estrogen and progesterone by the ovaries.

In hypothalamic amenorrhea, the production of gonadotropins is reduced due to a disruption of the hypothalamic-pituitary-ovarian axis, which can be caused by factors such as stress, weight loss, excessive exercise, or chronic illness.  

In this case, the history of difficult childbirth complicated by massive bleeding, along with the presence of symptoms such as weight loss, hair loss, lack of appetite, and depression, suggests that the patient may have experienced significant physical and emotional stress, which could have disrupted the normal function of the hypothalamic-pituitary-ovarian axis and led to the development of hypothalamic amenorrhea.  

Overproduction of prolactin (option A) can also cause amenorrhea, but typically presents with other symptoms such as galactorrhea (milk production) and breast tenderness. Reduced progesterone production (option C) can cause irregular periods but is unlikely to result in a complete absence of menstruation for 3 years.

Overproduction of estrogen (option D) can cause menstrual irregularities and other symptoms such as breast tenderness and mood changes, but is not typically associated with hypothalamic amenorrhea. Androgen overproduction (option E) can cause symptoms such as hirsutism and acne, but is also unlikely to cause a complete absence of menstruation for 3 years.   I hope this explanation helps! Let me know if you have any further questions.


2. vacationers in a forest glade drank alcohol, ate canned mushrooms, dried fish. The next day, two were hospitalized with impaired vision, swallowing, and breathing. The third had only a sharp general weakness, dry mouth. Two were healthy. A tick was found on the body of one healthy person. What is the most likely diagnosis? select one:

A. mushroom poisoning

b. Alcohol poisoning

C. Botulism

D. Lyme borreliosis

E. Tick-borne encephalitis


The correct answer is: Botulism

Explanation

The correct answer to the question is C. Botulism.   The scenario describes a group of vacationers who drank alcohol, ate canned mushrooms, and dried fish in a forest glade. The next day, two people were hospitalized with symptoms of impaired vision, swallowing, and breathing, while a third person experienced only general weakness and dry mouth. One healthy person was found to have a tick on their body.

These findings are consistent with a diagnosis of botulism, which is a rare but potentially life-threatening form of food poisoning caused by the bacterium Clostridium botulinum.   Botulism is typically caused by ingesting foods that have been contaminated with the botulinum toxin, which is produced by the Clostridium botulinum bacterium. Symptoms of botulism can include blurred vision, difficulty swallowing and breathing, dry mouth, muscle weakness, and paralysis. In severe cases, botulism can lead to respiratory failure and death.  

In this scenario, the symptoms of impaired vision, swallowing, and breathing in two of the individuals, along with general weakness and dry mouth in the third, are consistent with botulism. The tick found on one of the healthy individuals is unlikely to be related to the symptoms, as it is not a vector for botulism.   Mushroom poisoning (option A) can cause a range of symptoms depending on the type of mushroom ingested, but is unlikely to cause the specific symptoms described in this scenario.

Alcohol poisoning (option B) can cause symptoms such as vomiting, confusion, and respiratory depression, but is unlikely to cause the symptoms of impaired vision and swallowing seen in botulism. Lyme borreliosis (option D) is a bacterial infection transmitted by ticks that can cause a range of symptoms including fever, rash, and joint pain, but is not associated with the symptoms of botulism.

Tick-borne encephalitis (option E) is a viral infection transmitted by ticks that can cause symptoms such as fever, headache, and neurological symptoms, but is also unlikely to cause the symptoms described in this scenario.   In summary, the most likely diagnosis in this scenario is botulism, which is a rare but serious form of food poisoning caused by the botulinum toxin.   I hope this explanation helps! Let me know if you have any further questions.  


3. The victim was delivered to the burn department, who about 6 hours ago was hit by fire during a fire. On the body dry necrosis of gray-brown color, which covers 3/4 of its perimeter. In places, small vesicles with hemorrhagic contents, fragments of the epidermis. What will be the local treatment? select one:

A. Vesicle puncture

b. Chemical necrolysis

C. Necrectomy with xenoplasty

D. Decompression necrectomy

E. Necrectomy with autodermoplasty


The correct answer is: Decompression necrectomy

Explanation

The correct answer to the question is D. Decompression necrectomy.   The scenario describes a victim who suffered burns during a fire and has dry necrosis of gray-brown color covering 3/4 of their body, with small vesicles containing hemorrhagic contents and fragments of the epidermis.

This suggests a deep partial-thickness or full-thickness burn, which requires urgent medical attention.   Local treatment for burns depends on the severity and extent of the injury. In this case, the presence of dry necrosis and vesicles suggests that the burn is deep and may require surgical intervention.

The most appropriate local treatment in this scenario would be decompression necrectomy.   Decompression necrectomy involves the removal of necrotic tissue to reduce pressure and improve blood flow to the surrounding healthy tissue. This procedure is typically performed in cases of deep partial-thickness or full-thickness burns to prevent further tissue damage and promote wound healing. In this case, decompression necrectomy would be necessary to remove the necrotic tissue and prevent infection.  

Vesicle puncture (option A) may be appropriate in some cases to relieve pressure and prevent the spread of infection, but is not sufficient for the treatment of a deep partial-thickness or full-thickness burn. Chemical necrolysis (option B) is a treatment option for some types of burns, but is not typically used in cases of deep partial-thickness or full-thickness burns.

Necrectomy with xenoplasty (option C) and necrectomy with autodermoplasty (option E) are surgical procedures used in cases of extensive tissue loss, but may not be necessary in all cases of deep partial-thickness or full-thickness burns.   In summary, the most appropriate local treatment for the burn described in this scenario would be decompression necrectomy, which involves the removal of necrotic tissue to prevent further tissue damage and promote wound healing.   I hope this explanation helps! Let me know if you have any further questions.


4. The mother of a man suffering from epilepsy complains of periodic sudden seizures in her son of a sad and angry mood, during which he becomes agitated, prone to physical aggression, brutal actions. The attack lasts 5-10 minutes, after which the patient, exhausted, falls asleep. After awakening, he is depressed, sad, does not remember the circumstances of his past actions, or does not remember completely. What psychopathological state are we talking about? select one:

A. Pathological affect

b. Ambulatory automatism

C. Dysphoria

D. Dysthymia

E. epileptic delirium


The correct answer is: Dysphoria

Explanation

The correct answer to the question is C. Dysphoria.   The scenario describes a man with epilepsy who experiences periodic sudden seizures characterized by a sad and angry mood, agitation, physical aggression, and brutal actions.   These symptoms are followed by exhaustion and sleep, and upon awakening, the patient experiences depression, sadness, and memory loss. These findings are consistent with a diagnosis of dysphoria, which is a state of generalized dissatisfaction, restlessness, and agitation.  

Dysphoria is a common symptom of several psychiatric and neurological disorders, including epilepsy. In epilepsy, dysphoria can occur as part of a seizure or as an interictal symptom, meaning a symptom that occurs between seizures. Dysphoria can manifest as a range of symptoms, including irritability, agitation, anger, and sadness.  

The symptoms described in this scenario, including sudden mood changes, physical aggression, and memory loss, are consistent with dysphoria in the context of epilepsy. It is important to note that dysphoria can also be a side effect of some antiepileptic medications, so it is important to consider medication management as part of the treatment plan.  

Pathological affect (option A) refers to an abnormal emotional response that is out of proportion to the situation. Ambulatory automatism (option B) refers to a type of seizure characterized by complex, purposeful movements that the person is not aware of, and is not consistent with the symptoms described in this scenario.  

Dysthymia (option D) is a type of chronic depression that lasts for two years or more and is not characterized by sudden mood changes or physical aggression. Epileptic delirium (option E) is a rare condition characterized by confusion, disorientation, and hallucinations, and is not consistent with the symptoms described in this scenario.  

In summary, the most likely psychopathological state in this scenario is dysphoria, which is a state of generalized dissatisfaction, restlessness, and agitation commonly associated with epilepsy.   I hope this explanation helps! Let me know if you have any further questions.  


5. A patient suffering from atopic form of bronchial asthma was found to have the only allergen to the fur of a domestic dog +++. The carpets have been removed from the house, the apartment has been redecorated, air conditioning is being used. However, the patient has repeated attacks of suffocation every night, despite pathogenetic treatment. What long-term treatment tactics will potentially help the patient reduce the body’s susceptibility to the allergen? select one:

A. Antihistamine therapy

b. Exercise therapy classes according to Buteyko

C. Specific hyposensitization

D. Referral to speleotherapy

E. Continuation of previous treatment


The correct answer is: Specific hyposensitization

Explanation

The correct answer to the question is C. Specific hyposensitization.   The scenario describes a patient with atopic bronchial asthma whose only allergen is the fur of a domestic dog. Despite removing carpets, redecorating the apartment, and using air conditioning, the patient continues to experience repeated attacks of suffocation at night, despite receiving pathogenetic treatment. The most appropriate long-term treatment tactic in this scenario would be specific hyposensitization.  

Specific hyposensitization, also known as allergy immunotherapy, is a treatment that involves exposing the patient to small doses of the allergen over a period of time to reduce their sensitivity to the allergen. The goal of hyposensitization is to reduce the severity and frequency of allergic reactions and improve the patient’s quality of life. Hyposensitization is a long-term treatment approach that typically takes several years to complete.  

In this scenario, the patient’s only allergen is the fur of a domestic dog, and the patient continues to experience symptoms despite removing carpets, redecorating the apartment, and using air conditioning. This suggests that the patient may benefit from specific hyposensitization to reduce their sensitivity to the dog allergen and improve their asthma symptoms.  

Antihistamine therapy (option A) may be helpful in managing symptoms such as itching and runny nose, but is not a long-term treatment for reducing the body’s susceptibility to allergens. Exercise therapy classes according to Buteyko (option B) may be helpful in improving breathing and reducing the need for medication, but are not a specific treatment for reducing sensitivity to allergens.

Referral to speleotherapy (option D), which involves exposure to underground salt mines, has not been shown to be effective in the treatment of asthma. Continuation of previous treatment (option E) may be necessary, but is not sufficient for reducing the body’s susceptibility to the dog allergen.  

In summary, the most appropriate long-term treatment tactic for a patient with atopic bronchial asthma who is sensitive to the fur of a domestic dog is specific hyposensitization, which involves exposing the patient to small doses of the allergen over time to reduce their sensitivity and improve their symptoms.   I hope this explanation helps! Let me know if you have any further questions.


6. A 67-year-old woman suffering from hypertension suddenly had a headache at night, shortness of breath, which quickly turned into suffocation. Objectively: she is pale, there are sweat drops on her forehead, BP – 210/140 mm Hg, heart rate – 120/min., isolated dry rales over the lungs, moist rales in the lower parts. The shins are pasty. What emergency assistance is most appropriate in this case? select one:

A. Enalapril and furosemide IV

b. IV labetalol and furosemide

C. Digoxin and nitroglycerin IV

D. Nitroglycerin and furosemide IV

E. Nitroglycerin IV and capoten orally


The correct answer is: IV nitroglycerin and furosemide

Explanation

The correct answer to the question is D. IV nitroglycerin and furosemide.   The scenario describes a 67-year-old woman with hypertension who suddenly experiences a headache at night, followed by shortness of breath and suffocation. Upon examination, the patient is pale, has sweat drops on her forehead, a blood pressure of 210/140 mm Hg, a heart rate of 120/min, and rales in the lungs.

The patient’s shins are also pasty. Based on these symptoms, the most appropriate emergency assistance in this case would be IV nitroglycerin and furosemide.   IV nitroglycerin is a medication that dilates blood vessels and reduces blood pressure. It is commonly used in the emergency treatment of hypertension and heart failure. IV furosemide is a diuretic that helps remove excess fluid from the body, which can improve breathing and reduce blood pressure.  

In this scenario, the patient is experiencing a hypertensive emergency, which is a potentially life-threatening condition that requires immediate treatment to reduce blood pressure and prevent organ damage. The combination of IV nitroglycerin and furosemide is a commonly used treatment approach in hypertensive emergencies and is effective in rapidly reducing blood pressure and improving symptoms.  

Enalapril and furosemide IV (option A) and IV labetalol and furosemide (option B) are also treatment options for hypertensive emergencies, but may not be as effective as nitroglycerin and furosemide in rapidly reducing blood pressure. Digoxin and nitroglycerin IV (option C) and nitroglycerin IV and capoten orally (option E) are not appropriate treatments for hypertensive emergencies and may not effectively reduce blood pressure in this situation.  

In summary, the most appropriate emergency assistance for a patient with a hypertensive emergency characterized by shortness of breath, rales in the lungs, and high blood pressure is IV nitroglycerin and furosemide.   I hope this explanation helps! Let me know if you have any further questions.


7. A 34-year-old victim was delivered to a trauma center with an open fracture of the leg bones. On examination, the presence of bleeding is ascertained: blood flows out of the wound in a pulsating fountain. What therapeutic actions should be performed at this stage of medical care? select one:

A. Apply immobilization and transport the patient to the operating room

b. Stop the bleeding with a pressure bandage and transport the patient to the operating room

C. Transport the patient to the operating room

D. Apply a tourniquet to the thigh distal to the source of bleeding, and transport the patient to the operating room

E. Apply a tourniquet to the thigh proximal to the source of bleeding, and transport the patient to the operating room


The correct answer is: Place a tourniquet on the thigh proximal to the source of bleeding, and transport the patient to the operating room

Explanation

The correct answer to the question is E. Apply a tourniquet to the thigh proximal to the source of bleeding, and transport the patient to the operating room.   The scenario describes a 34-year-old victim with an open fracture of the leg bones who is bleeding profusely from the wound. In this situation, the most appropriate therapeutic action is to apply a tourniquet to the thigh proximal to the source of bleeding and transport the patient to the operating room.  

A tourniquet is a device used to stop bleeding by compressing the blood vessels and preventing blood flow to the affected area. In cases of severe bleeding, such as in this scenario, a tourniquet may be necessary to prevent the patient from going into shock and to save their life. The tourniquet should be applied as close to the source of bleeding as possible, and the patient should be transported to the operating room as soon as possible for definitive treatment.  

Applying immobilization and transporting the patient to the operating room (option A) or stopping the bleeding with a pressure bandage and transporting the patient to the operating room (option B) may not be sufficient to control the bleeding in this situation.

Transporting the patient to the operating room (option C) without controlling the bleeding first could lead to further blood loss and worsening of the patient’s condition. Applying a tourniquet to the thigh distal to the source of bleeding (option D) is not recommended, as it may not be effective in stopping the bleeding.  

In summary, the most appropriate therapeutic action for a patient with an open fracture of the leg bones who is bleeding profusely is to apply a tourniquet to the thigh proximal to the source of bleeding and transport the patient to the operating room for definitive treatment.   I hope this explanation helps! Let me know if you have any further questions.


8. A patient came to the doctor with complaints of fever up to $37.8^oC$, moderate sore throat for 3 days. Objectively: the maxillary lymph nodes are enlarged up to 3 cm. The palatine tonsils are hypertrophied, covered with a gray coating extending to the uvula and anterior palatine arches. When scraped, it bleeds. What is the most likely diagnosis? select one:

A. Angina Simanovsky-Vincent

b. Infectious mononucleosis

C. Agranulocytosis

D. Oropharyngeal diphtheria

E. Candidiasis of the oropharynx


The correct answer is: Oropharyngeal diphtheria

Explanation

The correct answer to the question is D. Oropharyngeal diphtheria.   The scenario describes a patient with a fever, sore throat, and enlarged maxillary lymph nodes. Upon examination, the patient’s palatine tonsils are hypertrophied and covered with a gray coating that extends to the uvula and anterior palatine arches, and bleeds when scraped.

These findings are consistent with a diagnosis of oropharyngeal diphtheria.   Oropharyngeal diphtheria is a bacterial infection caused by the bacterium Corynebacterium diphtheriae. It is characterized by the formation of a grayish-white pseudomembrane on the tonsils, pharynx, and/or nasal cavity.

The pseudomembrane can cause airway obstruction, leading to breathing difficulties and even death. Oropharyngeal diphtheria is a serious infection and requires immediate treatment with antibiotics and antitoxin.   Angina Simanovsky-Vincent (option A) is a type of tonsillitis caused by a combination of bacteria, including Fusobacterium necrophorum and other anaerobic bacteria. It typically presents with severe sore throat, fever, and enlarged lymph nodes.

However, it does not cause the formation of a pseudomembrane, which is a key feature of oropharyngeal diphtheria.   Infectious mononucleosis (option B) is a viral infection caused by the Epstein-Barr virus. It typically presents with fever, sore throat, and enlarged lymph nodes, but does not cause the formation of a pseudomembrane.   Agranulocytosis (option C) is a condition in which the body produces too few white blood cells, which can lead to an increased risk of infections.

It does not cause the formation of a pseudomembrane in the oropharynx.   Candidiasis of the oropharynx (option E) is a fungal infection caused by Candida species. It typically presents with white patches on the tongue and the inside of the cheeks, but does not cause the formation of a pseudomembrane.  

In summary, the most likely diagnosis for a patient with a fever, sore throat, and the formation of a grayish-white pseudomembrane on the tonsils is oropharyngeal diphtheria, which is a serious bacterial infection that requires immediate treatment with antibiotics and antitoxin.   I hope this explanation helps! Let me know if you have any further questions.


9. A 30-year-old patient applied with complaints of milk secretion from the mammary glands, absence of menstruation for 5 months. Childbirth – 1, physiological, four years ago. Breast development is normal. Bimanual examination revealed a decrease in the uterus and normal size of the ovaries. MRI showed no brain pathology. Thyrotropic hormone – within normal limits. The level of prolactin in the blood serum is increased. What is the most likely diagnosis? select one:

A. polycystic ovary syndrome

b. Hyperprolactinemia

C. pituitary adenoma

D. Sheehan syndrome

E. Hypothyroidism


The correct answer is: Hyperprolactinemia

Explanation

The correct answer to the question is B. Hyperprolactinemia.   The scenario describes a 30-year-old patient with milk secretion from the mammary glands, absence of menstruation for 5 months, and a history of one physiological childbirth four years ago. Breast development is normal, and bimanual examination revealed a decrease in the uterus and normal size of the ovaries.

MRI showed no brain pathology, and thyrotropic hormone was within normal limits. The level of prolactin in the blood serum is increased. These findings are consistent with a diagnosis of hyperprolactinemia.   Hyperprolactinemia is a condition characterized by elevated levels of prolactin in the blood. Prolactin is a hormone that stimulates milk production in the mammary glands.

In women, elevated prolactin levels can cause milk secretion from the breasts, amenorrhea (absence of menstruation), and infertility. Hyperprolactinemia can be caused by a variety of factors, including pituitary tumors, medications, and other medical conditions.   In this scenario, the most likely cause of hyperprolactinemia is a pituitary tumor, which can cause an increase in prolactin secretion. Pituitary adenomas (option C) are a type of tumor that can cause hyperprolactinemia and may require surgical treatment.

Sheehan syndrome (option D) is a rare condition that occurs when the pituitary gland is damaged due to severe bleeding or shock during childbirth, causing a decrease in pituitary hormone production. Polycystic ovary syndrome (option A) is a hormonal disorder that can cause irregular menstruation, but does not typically cause elevated prolactin levels. Hypothyroidism (option E) can also cause menstrual irregularities and elevated prolactin levels, but the scenario states that thyrotropic hormone levels were within normal limits.  

In summary, the most likely diagnosis for a patient with milk secretion from the mammary glands, absence of menstruation, and an increase in prolactin levels is hyperprolactinemia, which can be caused by a pituitary tumor or other factors. Treatment may include medication to lower prolactin levels or surgery to remove the pituitary tumor.   I hope this explanation helps! Let me know if you have any further questions.


10. A 52-year-old patient has lesions of the mucous membrane in the corners of the mouth with the formation of cracks, erosions and ulcers, vertical cracks on the lips when they are closed (cheilosis), a change in the tongue (glossitis), angular stomatitis, seborrheic peeling of the skin around the mouth and on the wings of the nose , pericorneal injection. The above symptoms are typical for: select one:

A. C-hypovitaminosis

b. PP-hypovitaminosis

C. A-hypovitaminosis

D. B₁-hypovitaminosis

E. B₂-hypovitaminosis


The correct answer is: B₂-hypovitaminosis

Explanation

The correct answer to the question is E. B₂-hypovitaminosis.   The scenario describes a 52-year-old patient with lesions of the mucous membrane in the corners of the mouth, cheilosis, glossitis, angular stomatitis, seborrheic peeling of the skin around the mouth and on the wings of the nose, and pericorneal injection.

These symptoms are typical for B₂-hypovitaminosis, which is a deficiency of vitamin B2 (riboflavin).   Riboflavin is a water-soluble vitamin that is necessary for the proper functioning of many enzymes in the body. A deficiency of vitamin B2 can lead to a variety of symptoms, including skin and mucous membrane lesions, cheilosis (vertical cracks on the lips), glossitis (inflammation of the tongue), and seborrheic peeling of the skin around the mouth and nose. Pericorneal injection, or redness around the cornea, can also occur in severe cases.  

Hypovitaminosis C (option A) is a deficiency of vitamin C, which can lead to scurvy. Symptoms of scurvy include fatigue, weakness, joint and muscle pain, and skin and mucous membrane lesions, but cheilosis and pericorneal injection are not commonly seen.  

Hypovitaminosis PP (option B) is a deficiency of vitamin PP, also known as niacin or vitamin B3. Symptoms of niacin deficiency include skin lesions, diarrhea, dementia, and other neurological symptoms, but cheilosis and pericorneal injection are not commonly seen.  

Hypovitaminosis A (option C) is a deficiency of vitamin A, which can lead to night blindness, dry eyes, and skin and mucous membrane lesions, but cheilosis and pericorneal injection are not commonly seen.   Hypovitaminosis B₁ (option D) is a deficiency of vitamin B1, also known as thiamine.

Symptoms of thiamine deficiency include fatigue, muscle weakness, and neurological symptoms, but cheilosis and pericorneal injection are not commonly seen.   In summary, the symptoms described in the scenario are typical for B₂-hypovitaminosis, which is a deficiency of vitamin B2 (riboflavin). Treatment involves increasing intake of foods rich in vitamin B2 or taking riboflavin supplements.   I hope this explanation helps! Let me know if you have any further questions.


11. The patient went to the clinic with complaints of weight gain, chilliness, swelling, dry skin, drowsiness, difficulty concentrating. Objectively: height is 165 cm, weight is 90 kg, female body proportions, body temperature is 35.8°C, heart rate is 58/min., blood pressure is 105/60 mm Hg. Heart sounds are weakened, bradycardia. Other internal organs are unchanged. The thyroid gland is not palpated. There is a release of drops of milk from the mammary glands. A hormonal study found an increase in TSH and prolactin levels, a decrease in T4. Which of the reasons led to the formation of obesity? select one:

A. Adiposogenital dystrophy

b. hypopituitarism

C. Secondary hypothyroidism

D. Prolactinoma

E. Primary hypothyroidism


The correct answer is: Primary hypothyroidism

Explanation

The patient’s symptoms and objective findings are consistent with hypothyroidism, which is a condition where the thyroid gland is not producing enough thyroid hormones. The decreased T4 levels and increased TSH levels found in the hormonal study further support this diagnosis.

Hypothyroidism can lead to weight gain, chilliness, dry skin, drowsiness, and difficulty concentrating, as well as bradycardia and weakened heart sounds. The release of drops of milk from the mammary glands is also consistent with high prolactin levels, which can be caused by hypothyroidism.  

Adiposogenital dystrophy, also known as Frohlich’s syndrome, is a rare condition that can cause obesity and sexual dysfunction, but it is not associated with hypothyroidism. Hypopituitarism can cause hormonal imbalances, but it is not the most likely cause of the patient’s symptoms and objective findings.

Secondary hypothyroidism, which is caused by a problem with the pituitary gland, can cause similar symptoms to primary hypothyroidism, but it is less common. Prolactinoma, a type of pituitary tumor that produces prolactin, can cause high prolactin levels and milk production, but it is not likely to be the primary cause of the patient’s symptoms and objective findings.


12. After inhaling the aerosol, the patient developed widespread rashes all over the skin, as well as on the oral mucosa, after a few hours. Rashes are represented by a macular rash, on the mucous membranes – single blisters with serous contents. There was an increase in temperature up to 38.8°C, a single vomiting. Specify the preliminary diagnosis: select one:

A. Scabies

b. Toxicoderma

C. Syphilis

D. True eczema

E. Simple contact dermatitis


The correct answer is: Toxicoderma

Explanation

The patient’s symptoms, including the widespread macular rash, blisters on the oral mucosa, and fever, are consistent with a condition called toxicoderma. Toxicoderma is a general term used to describe a skin reaction caused by exposure to a medication, chemical, or toxin.

The development of symptoms after inhaling an aerosol suggests that the patient may have had an allergic reaction to a substance in the aerosol.   Scabies is a parasitic infection that can cause a rash, but it typically presents as small, raised bumps rather than a macular rash.

Syphilis can also cause a rash, but it typically presents as a painless, non-itchy rash on the palms and soles of the feet, which is different from the macular rash described in the prompt. True eczema is a chronic skin condition characterized by dry, itchy patches of skin, and it is not typically associated with blisters or fever. Simple contact dermatitis is a common skin reaction to an irritant or allergen, but it usually presents as a localized rash rather than a widespread rash with blisters.  

It is important to note that this is a preliminary diagnosis based on the information provided in the prompt. Further evaluation, such as a detailed medical history and physical examination, may be necessary to confirm the diagnosis and identify the underlying cause of the toxicoderma.


13. A 27-year-old woman notes discharge from the genital tract with an unpleasant odor, pain in the lower abdomen, fever. The above complaints appeared 2 days ago. In the anamnesis: a week ago, the woman underwent a surgical termination of pregnancy for a period of 8 weeks. Inspection in the mirrors: the cervix is clean, from the external pharynx discharge with an unpleasant odor. Vaginal examination: uterus in anteflexio, mobile, painful, slightly enlarged. Appendages without features. Provisional diagnosis: select one:

A. Appendicitis

b. Enterocolitis

C. Post-abortion endometritis

D. Acute respiratory disease

E. Salpingoophoritis


The correct answer is: Post-abortion endometritis

Explanation

The patient’s symptoms, including discharge from the genital tract with an unpleasant odor, pain in the lower abdomen, and fever, are consistent with a diagnosis of post-abortion endometritis. Endometritis is an infection of the lining of the uterus, and it can occur after a surgical or spontaneous abortion.

The presence of an unpleasant odor from the discharge suggests that there may be an infection present. The fact that the woman underwent a surgical termination of pregnancy a week ago also supports this diagnosis.   Appendicitis and enterocolitis are conditions that typically present with abdominal pain, but they do not usually cause genital discharge or fever. Acute respiratory disease is a respiratory condition that is unlikely to cause the symptoms described in the prompt.

Salpingoophoritis, also known as pelvic inflammatory disease, is a condition that can cause lower abdominal pain and fever, but it typically presents with vaginal discharge that is different in color and consistency from the discharge described in the prompt.  

It is important to note that this is a preliminary diagnosis based on the information provided in the prompt. Further evaluation, such as a detailed medical history and physical examination, may be necessary to confirm the diagnosis and identify the most appropriate treatment plan for the patient.  


14. A 62-year-old woman was brought to the emergency department with complaints of severe burning pain behind the sternum, suffocation. A history of 10 years of hypertension. Objectively: the state of moderate severity, pale skin, cyanosis of the lips, vesicular breathing over the lungs. Heart sounds are muffled, rhythmic. Accent II tone over the aorta. Blood pressure – 210/120 mm Hg. Heart rate = pulse – 76/min. On the ECG: an increase in the ST segment in I, aVL, V5-V6 leads. Most likely diagnosis: select one:

A. Hypertensive crisis complicated by acute left ventricular failure

b. Hypertensive crisis complicated by unstable angina pectoris

C. BODY

D. Uncomplicated hypertensive crisis

E. Hypertensive crisis complicated by acute myocardial infarction


The correct answer is: Hypertensive crisis complicated by acute myocardial infarction

Explanation

The patient’s symptoms, including severe burning pain behind the sternum, suffocation, and cyanosis of the lips, along with the objective findings of muffled heart sounds and an accentuated II tone over the aorta, suggest a diagnosis of hypertensive crisis complicated by acute myocardial infarction.

The elevated blood pressure of 210/120 mm Hg, along with the ECG findings of an ST segment elevation in multiple leads, further support this diagnosis.   Unstable angina pectoris can cause chest pain and may be associated with ECG changes, but it is less likely to cause the severity of symptoms and objective findings described in the prompt.

A hypertensive crisis complicated by acute left ventricular failure is possible, but the absence of significant pulmonary congestion and the vesicular breathing over the lungs make this less likely. BODY (brain attack, obstruction of the pulmonary artery, dissection of the aorta, and other acute conditions) is a mnemonic used to remember potential causes of acute chest pain, but it is not a specific diagnosis.

An uncomplicated hypertensive crisis can cause high blood pressure, but it is not typically associated with the severe chest pain and ECG changes seen in the prompt.   It is important to note that this is a preliminary diagnosis based on the information provided in the prompt. Further evaluation, such as laboratory tests and imaging studies, may be necessary to confirm the diagnosis and determine the most appropriate treatment plan for the patient.  


15. A 25-year-old patient with frequent bleeding from the genital tract was delivered to the gynecological department. Desirable pregnancy for 12 weeks. Over the past 3 days, she was worried about pain in the lower abdomen, which took on a paroxysmal character, bleeding appeared. The skin is pale, the pulse is 88/min., blood pressure is 100/60 mm Hg, body temperature is 36.8°C. During vaginal examination: the uterus is enlarged up to 11 weeks of pregnancy, the cervical canal passes 1 finger, parts of the fetal egg are determined in the canal, bloody, frequent discharge. What is the most likely diagnosis? select one:

A. Term pregnancy, term delivery

b. Pregnancy 12 weeks, threatened miscarriage

C. Menstrual disorders, hyperpolymenorrhea

D. Menstrual disorders, amenorrhea

E. Pregnancy 12 weeks, spontaneous miscarriage in progress


The correct answer is: Pregnant 12 weeks, spontaneous miscarriage in progress

Explanation

The patient’s symptoms, including frequent bleeding from the genital tract, pain in the lower abdomen, and the presence of parts of the fetal egg in the cervical canal, suggest a diagnosis of spontaneous miscarriage in progress. The enlarged uterus up to 11 weeks of pregnancy and the bloody, frequent discharge further support this diagnosis.  

Term pregnancy, term delivery is not consistent with the patient’s symptoms and objective findings, as the presence of parts of the fetal egg in the cervical canal suggests that the pregnancy is not viable. Menstrual disorders, such as hyperpolymenorrhea and amenorrhea, are not likely to cause the symptoms described in the prompt, including the enlarged uterus and the presence of parts of the fetal egg.

Threatened miscarriage is a possibility, but the presence of parts of the fetal egg in the cervical canal suggests that the miscarriage is in progress.   It is important to note that this is a preliminary diagnosis based on the information provided in the prompt. Further evaluation, such as an ultrasound examination or laboratory tests, may be necessary to confirm the diagnosis and determine the most appropriate treatment plan for the patient.


16. A 37-year-old patient has episodes of loss of consciousness, shortness of breath during physical exertion, periodic interruptions in the work of the heart. The patient’s father died suddenly at the age of 45. Objectively: Ps90/min., BP- 140/90 mm Hg. With Echo-CS: EF – 49%, a significant increase in the thickness of the myocardium of the left ventricle and interventricular septum. What drug is indicated for treatment? select one:

A. Hydrochlorothiazide

b. Furosemide

C. Fenigidin

D. bisoprolol

E. Enalapril


The correct answer is: Bisoprolol

Explanation

The patient’s symptoms, including episodes of loss of consciousness, shortness of breath during physical exertion, and periodic interruptions in the work of the heart, along with the objective findings of elevated blood pressure and significant increase in the thickness of the myocardium of the left ventricle and interventricular septum on Echo-CS, suggest a diagnosis of hypertrophic cardiomyopathy.  

Bisoprolol is a beta-blocker that is indicated for the treatment of hypertrophic cardiomyopathy. Beta-blockers can help to reduce the workload of the heart, improve heart function, and reduce symptoms, such as shortness of breath and palpitations. They can also help to prevent arrhythmias and sudden death in patients with hypertrophic cardiomyopathy.  

Hydrochlorothiazide and furosemide are diuretics that are used to treat hypertension and heart failure, but they are not typically used for hypertrophic cardiomyopathy. Fenigidin is a calcium channel blocker that is used to treat hypertension, but it is not typically used for hypertrophic cardiomyopathy.

Enalapril is an ACE inhibitor that is used to treat hypertension and heart failure, but it is not typically used for hypertrophic cardiomyopathy.   It is important to note that this is a preliminary diagnosis based on the information provided in the prompt. Further evaluation, such as additional diagnostic testing and a detailed medical history, may be necessary to confirm the diagnosis and determine the most appropriate treatment plan for the patient.  


17. After an ischemic stroke caused by cardioembolism, a patient with atrial fibrillation is prescribed as a means of secondary prevention: select one:

A. Nootropic drugs

b. Aspirin or clopidogrel

C. calcium antagonists

D. β-blockers

E. Oral anticoagulants


The correct answer is: Oral anticoagulants

Explanation

The patient’s history of atrial fibrillation and cardioembolic stroke suggests that they are at a high risk of developing another stroke. Oral anticoagulants, such as warfarin or direct oral anticoagulants (DOACs), are the recommended therapy for secondary prevention of stroke in patients with atrial fibrillation.  

Nootropic drugs, calcium antagonists, and β-blockers are not typically used for secondary prevention of stroke in patients with atrial fibrillation. Aspirin or clopidogrel may be used in some cases, but they are less effective than oral anticoagulants and are generally reserved for patients who are unable to take anticoagulants due to bleeding risk or other concerns.  

It is important to note that the choice of anticoagulant therapy and dosing should be individualized based on the patient’s specific clinical characteristics and bleeding risk. Regular monitoring of anticoagulation status is also important to ensure that the patient is maintained within the target therapeutic range.  


18. A 65-year-old patient with acute anterior myocardial infarction developed an asthma attack. On examination: diffuse cyanosis. In the lungs there are a large number of wet rales of various sizes. Heart rate – 100/min. BP – 160/100 mm Hg. What complication has developed in the patient? select one:

A. Pulmonary edema

b. Hypertensive crisis

C. Rupture of the interventricular septum

D. TELA E. Cardiogenic shock


The correct answer is: Pulmonary edema

Explanation

The patient’s symptoms, including an asthma attack, cyanosis, and wet rales of various sizes in the lungs, along with the objective findings of an elevated heart rate and blood pressure, suggest a diagnosis of pulmonary edema.   Pulmonary edema is a common complication of acute myocardial infarction, particularly in cases of anterior infarction. It occurs when there is an accumulation of fluid in the lungs, which can cause difficulty breathing, coughing, and wheezing. The elevated heart rate and blood pressure may be a compensatory response to the decreased cardiac output caused by the myocardial infarction.  

Hypertensive crisis is a possibility, but the presence of wet rales in the lungs and the diffuse cyanosis make pulmonary edema a more likely diagnosis. Rupture of the interventricular septum is a rare complication of acute myocardial infarction that can cause hypotension and shortness of breath, but it is not typically associated with an asthma attack or wet rales in the lungs. TELA (thromboembolism of the pulmonary artery) is a potential complication of acute myocardial infarction, but it typically presents with sudden onset of dyspnea, chest pain, and hemoptysis.

Cardiogenic shock can occur as a complication of acute myocardial infarction, but it is typically characterized by hypotension and decreased cardiac output, rather than elevated blood pressure and pulmonary edema.   It is important to note that this is a preliminary diagnosis based on the information provided in the prompt. Further evaluation, such as laboratory tests and imaging studies, may be necessary to confirm the diagnosis and determine the most appropriate treatment plan for the patient.


19. A 26-year-old patient is undergoing regular examination by a gynecologist. Makes no complaints. PV Uterus in anteflexio, not enlarged, firm, mobile, painless. To the left of the uterus, in the region of the appendages, a volumetric formation is palpable, mobile, which is displaced separately from the uterus, painless. Right appendages are not defined. What additional research method will be informative to clarify the diagnosis? select one:

A. Colonoscopy

b. Colposcopy

C. Ultrasound of the pelvic organs

D. Examination for urogenital infection

E. Metrosalpingography


The correct answer is: Ultrasound of the pelvic organs

Explanation

The presence of a mobile, painless, palpable mass in the left adnexal region, separate from the uterus, suggests a diagnosis of an ovarian cyst or tumor. To confirm the diagnosis and determine the nature and characteristics of the mass, an ultrasound of the pelvic organs would be the most informative additional diagnostic test.  

Ultrasound is a safe, non-invasive imaging modality that can provide detailed information about the size, location, and characteristics of ovarian masses. It can help to differentiate between benign and malignant masses and guide further management, such as observation, surgery, or other interventions.  

Colonoscopy is a test used to examine the colon and rectum and is not typically indicated for the evaluation of ovarian masses. Colposcopy is a test used to examine the cervix and is not typically indicated for the evaluation of adnexal masses. Examination for urogenital infection may be necessary in some cases, but it is not likely to be informative in clarifying the diagnosis of an adnexal mass.

Metrosalpingography is a test used to evaluate the fallopian tubes and is not typically indicated for the evaluation of ovarian masses.


20. A 32-year-old woman complains of general weakness, subfebrile condition for four months, back pain, dysuria. History: frequent acute respiratory diseases (ARI), hypothermia, lowcalorie diets, pulmonary tuberculosis in childhood. In urine: pH-4.8, leukocyturia, hematuria. In the blood: leukocytosis, lymphocytosis, increased erythrocyte sedimentation rate. The conclusion of the urographic examination: expansion of the pelvicalyceal system of both kidneys, foci of calcification in the projection of the parenchyma of the right kidney. What is the most likely diagnosis? select one:

A. right kidney cyst

b. Glomerulonephritis acute

C. Right kidney cancer

D. Nephrotuberculosis

E. Chronic pyelonephritis


The correct answer is: Nephrotuberculosis

Explanation

The patient’s symptoms, including general weakness, subfebrile condition, back pain, dysuria, and hematuria, along with the history of pulmonary tuberculosis in childhood, suggest a diagnosis of nephrotuberculosis. The urographic examination findings of expansion of the pelvicalyceal system and foci of calcification in the parenchyma of the right kidney also support this diagnosis.  

Nephrotuberculosis is a rare form of tuberculosis that affects the kidneys. It can cause a range of symptoms, including hematuria, dysuria, back pain, and general malaise. The disease can cause scarring and damage to the renal tissue, which can lead to chronic kidney disease and renal failure if left untreated.  

Right kidney cyst, glomerulonephritis acute, right kidney cancer, and chronic pyelonephritis are possible differential diagnoses but are less likely given the patient’s symptoms and urographic examination findings. An accurate diagnosis will require further evaluation, such as a biopsy or additional imaging studies.  

It is important to note that tuberculosis is a serious infectious disease that requires prompt diagnosis and treatment with appropriate anti-tuberculosis medications. Delayed or inadequate treatment can lead to significant morbidity and mortality.  


21. A 23-year-old patient with type I diabetes mellitus developed nausea and vomiting in the second week of community-acquired pneumonia. In the evening she lost consciousness. Hospitalized. Objectively: the skin is dry and pale. Breathing is noisy, the tongue is dry, with a layer of brown color. Heart rate – 129/min., BP – 85/50 mm Hg. Does not respond to palpation of the abdomen. The liver is +3 cm. The reaction to acetone is sharply positive, blood glucose is 26 mmol/l. Provisional diagnosis: select one:

A. Ketoacidotic coma

b. hepatic coma

C. Hyperosmolar coma

D. Lactic coma

E. Infectious-toxic shock


The correct answer is: Ketoacidotic coma

Explanation

The patient’s symptoms, including nausea, vomiting, dry and pale skin, dry and brown tongue, tachycardia, hypotension, positive reaction to acetone, and elevated blood glucose level, suggest a diagnosis of ketoacidotic coma in the setting of type I diabetes mellitus and community-acquired pneumonia.  

Ketoacidotic coma is a life-threatening complication of uncontrolled diabetes mellitus, which can occur as a result of a lack of insulin or an excess of counter-regulatory hormones, such as glucagon and cortisol. This leads to the breakdown of fats and the production of ketones, which can cause metabolic acidosis and dehydration.  

Hepatic coma, hyperosmolar coma, lactic coma, and infectious-toxic shock are less likely given the patient’s symptoms and medical history. Hepatic coma typically occurs in the setting of acute or chronic liver failure, which is not suggested by the physical examination findings.

Hyperosmolar coma is a complication of uncontrolled diabetes mellitus, but it typically presents with hyperosmolarity and dehydration, rather than ketoacidosis. Lactic coma can occur in the setting of metabolic acidosis, but it is typically associated with tissue hypoxia and anaerobic metabolism.

Infectious-toxic shock is a possibility given the patient’s history of community-acquired pneumonia, but the presence of ketoacidosis and hyperglycemia suggest a primary metabolic disorder.   Immediate treatment for ketoacidotic coma includes insulin therapy, fluid and electrolyte replacement, and correction of acid-base imbalances. It is important to note that this is a medical emergency that requires prompt recognition and intervention to prevent serious complications and death.


22. A 9-month-old child has a high fever, cough, shortness of breath. He fell ill 5 days ago after contact with patients with acute respiratory viral infections. Objectively: the child’s condition is serious. Temperature $38^oC$, cyanosis of the nasolabial triangle. Respiratory rate – 54/min., swelling of the wings of the nose during breathing. Percutere: above the lungs, a shortening of the sound on the right below the angle of the scapula, above other areas – a tympanic tone of the sound. On auscultation – fine bubbling wet rales on both sides, more on the right. What is the most likely diagnosis? select one:

A. Acute bronchiolitis

b. Acute laryngotracheitis

C. Acute bronchitis

D. Acute pneumonia

E. SARS


The correct answer is: Acute pneumonia

Explanation

The patient’s symptoms, including high fever, cough, shortness of breath, and cyanosis, along with the physical examination findings of tachypnea, nasal flaring, and wet rales on auscultation, suggest a diagnosis of acute pneumonia.   Acute pneumonia is a common respiratory tract infection in children, particularly those under the age of 2 years.

It is typically caused by bacterial or viral pathogens and can present with a range of symptoms, including fever, cough, shortness of breath, and chest pain. Physical examination findings may include tachypnea, nasal flaring, and crackles or wheezing on auscultation.  

Acute bronchiolitis, acute laryngotracheitis, acute bronchitis, and SARS are possible differential diagnoses but are less likely given the patient’s symptoms and physical examination findings. Acute bronchiolitis typically presents with wheezing, and acute laryngotracheitis (also known as croup) typically presents with a barking cough and inspiratory stridor.

Acute bronchitis may present with a cough and wheezing, but it typically does not cause tachypnea or cyanosis. SARS is a severe respiratory illness caused by a coronavirus and is less likely given the patient’s age and contact history.   It is important to note that prompt recognition and treatment of pneumonia are important to prevent serious complications, such as respiratory failure and sepsis. Treatment may include antibiotics, oxygen therapy, and supportive care.  


23. A 37-year-old patient, a typesetter in a printing house, complains of rapid fatigue, attacks of paroxysmal pain in the abdomen, weakness in the hands, hands drooping. When examining the neurological status, hypotrophy of the muscles of the forearms is noted. Carporadial reflexes are sharply reduced. Sensitivity is not broken. A dark blue border is noted along the edges of the gums. What neurological pathology does the patient have? select one:

A. Guillain-Barré polyradiculoneuritis

b. Shingles

C. Lead polyneuropathy

D. Shoulder plexitis

E. Ulnar nerve neuropathy


The correct answer is: Lead polyneuropathy

Explanation

The patient’s symptoms, including rapid fatigue, abdominal pain, weakness in the hands, hypotrophy of the forearm muscles, reduced carporadial reflexes, and a dark blue border along the edges of the gums, suggest a diagnosis of lead polyneuropathy.   Lead polyneuropathy is a neurological disorder caused by exposure to lead, which can occur in occupational settings or through environmental contamination. It can cause a range of symptoms, including fatigue, abdominal pain, weakness in the hands and forearms, and reduced deep tendon reflexes.

The blue line along the gums, known as a Burton’s line, is a classic sign of chronic lead poisoning.   Guillain-Barré polyradiculoneuritis, shingles, shoulder plexitis, and ulnar nerve neuropathy are possible differential diagnoses but are less likely given the patient’s symptoms and findings.

Guillain-Barré polyradiculoneuritis typically presents with ascending weakness and sensory loss, and shingles typically presents with a painful rash in a dermatomal distribution. Shoulder plexitis and ulnar nerve neuropathy may cause weakness and sensory loss in the upper extremities, but they typically do not cause abdominal pain or a Burton’s line.  

It is important to note that lead poisoning is a serious condition that can affect multiple organ systems, including the central nervous system, kidneys, and hematopoietic system. Treatment may include removal of the source of lead exposure, chelation therapy, and supportive care for neurological symptoms.  


24. After hitting the head, a patient developed cerebral symptoms, nausea, vomiting, focal symptoms – hemihyperreflexia S>D, hemihypesthesia on the left, severe meningeal syndrome. An X-ray of the skull revealed no pathology. CT revealed no pathology. What method of examination will clarify and confirm the diagnosis? select one:

A. Angiography

b. Lumbar puncture

C. Echo encephalography

D. Pneumoencephalography

E. Electroencephalography


The correct answer is: Lumbar Puncture

Explanation

The patient’s symptoms, including cerebral symptoms, nausea, vomiting, focal symptoms of hemihyperreflexia and hemihypesthesia, and severe meningeal syndrome, suggest a diagnosis of traumatic brain injury with possible intracranial hemorrhage or cerebrospinal fluid (CSF) leakage. The negative X-ray and CT results suggest that a more sensitive diagnostic test may be needed to confirm the diagnosis.  

Lumbar puncture (also known as a spinal tap) is a diagnostic procedure that involves the insertion of a needle into the subarachnoid space in the lower back, where CSF can be obtained for analysis. It can be used to confirm the presence of intracranial hemorrhage or CSF leakage, which may not be visible on imaging studies in the early stages after injury.

The presence of blood or an elevated protein level in the CSF can indicate a traumatic injury and help guide further management.   Angiography, echo encephalography, and pneumoencephalography are less likely to be helpful in this case, as they are not typically indicated for the evaluation of traumatic brain injury or intracranial hemorrhage.

Electroencephalography may be useful in some cases to evaluate for seizures or other neurological abnormalities, but it is not likely to provide information about the presence of intracranial hemorrhage or CSF leakage.  

It is important to note that traumatic brain injury is a serious condition that requires prompt recognition and management to prevent serious complications, including intracranial hemorrhage, cerebral edema, and seizures. Immediate treatment may include monitoring, supportive care, and surgical intervention if necessary.


25. As a result of improper storage, sprouted or green potatoes have a bitter taste. What toxic substance contained in such a potato can cause food poisoning?

a. Helvelic acid

b. solanine

c. Muscarine

d. Fazin

e. Muscaridine


The correct answer is: Solanine

Explanation

The toxic substance contained in sprouted or green potatoes that can cause food poisoning is solanine. Solanine is a glycoalkaloid that is naturally present in potatoes and other members of the nightshade family, such as tomatoes and eggplants. It is produced by the plant as a natural defense mechanism against insects, fungi, and other predators.  

When potatoes are exposed to light or stored improperly, they may begin to sprout or turn green, indicating an increased level of solanine. Ingestion of solanine can cause a range of symptoms, including nausea, vomiting, diarrhea, stomach cramps, headache, and dizziness. In severe cases, it can cause respiratory depression, seizures, and even death.  

Helvellic acid, muscarine, fazin, and muscaridine are also toxic substances, but they are not typically associated with potato poisoning. Helvellic acid is found in certain mushrooms and can cause liver damage. Muscarine is found in certain mushrooms and can cause cholinergic symptoms such as sweating, salivation, and diarrhea.

Fazin is found in fava beans and can cause hemolytic anemia in individuals with glucose-6-phosphate dehydrogenase deficiency. Muscaridine is a synthetic compound that has cholinergic activity and is not found in natural sources.


26. A 34-year-old patient on the 3rd day of treatment of acute otitis media with ceftriaxone at a daily dose of 2 grams developed loose stools 5-6 times a day. Fecal mass without impurities of blood, mucus. Body temperature – 36.6 ° C. Gregersen’s reaction to occult blood is negative. Bacteriological examination of feces did not reveal pathogenic microorganisms. What is the most common cause of diarrhea? select one:

A. Intestinal dysbiosis

b. Antibiotic-associated diarrhea

C. Ulcerative colitis

D. bacterial overgrowth syndrome

E. Crohn’s disease


The correct answer is: Antibiotic-associated diarrhea

Explanation

The most likely cause of the patient’s diarrhea is antibiotic-associated diarrhea (AAD). AAD is a common side effect of antibiotic therapy and is caused by disruption of the normal gut microbiota, leading to overgrowth of pathogenic bacteria such as Clostridium difficile.

AAD can present with loose stools, abdominal cramping, and sometimes fever, and typically resolves within a few days to weeks after completion of antibiotic therapy.   Intestinal dysbiosis, ulcerative colitis, bacterial overgrowth syndrome, and Crohn’s disease are possible differential diagnoses but are less likely given the patient’s symptoms and findings.

Intestinal dysbiosis refers to an imbalance of the gut microbiota and can cause a range of gastrointestinal symptoms, but it typically does not cause diarrhea in the absence of other pathogenic organisms. Ulcerative colitis and Crohn’s disease are chronic inflammatory bowel diseases that can cause diarrhea, but they typically present with other symptoms such as weight loss, rectal bleeding, and abdominal pain. Bacterial overgrowth syndrome can cause diarrhea and other gastrointestinal symptoms, but it is typically associated with underlying conditions such as diabetes or gastrointestinal surgery.  

It is important to note that AAD can be prevented or minimized by using antibiotics judiciously and taking probiotics during and after antibiotic therapy. In most cases, AAD is self-limited and does not require specific treatment, but in severe cases, antibiotics such as metronidazole or vancomycin may be necessary.  


27. A 5-year-old child fell ill acutely with a sharp rise in body temperature to 39.5°C, severe chills, weakness, lethargy, pallor of the skin, and headache. After 8 hours, a hemorrhagic rash appeared on the skin of the buttocks and lower extremities. The child is lethargic, the temperature has dropped, blood pressure is 80/40 mm Hg, respiratory rate is 28-30/min., diuresis is reduced. Establish a preliminary diagnosis: select one:

A. Measles

b. Thrombocytopenic purpura

C. Reye’s syndrome

D. Meningococcemia

E. Hemorrhagic vasculitis


The correct answer is: Meningococcemia

Explanation

The patient’s symptoms, including acute onset of high fever, chills, weakness, lethargy, headache, and hemorrhagic rash, suggest a diagnosis of meningococcemia. Meningococcemia is a serious bacterial infection caused by Neisseria meningitidis, which can cause sepsis and meningitis. It is more common in children and young adults and can be rapidly fatal if left untreated.  

Thrombocytopenic purpura, Reye’s syndrome, hemorrhagic vasculitis, and measles are possible differential diagnoses but are less likely given the patient’s symptoms and findings. Thrombocytopenic purpura is a condition characterized by low platelet counts and bleeding, but it typically does not cause fever or sepsis. Reye’s syndrome is a rare but serious condition that can occur in children with viral illnesses who are treated with aspirin, but it typically does not cause a hemorrhagic rash.

Hemorrhagic vasculitis (also known as Henoch-Schonlein purpura) can cause a purpuric rash and abdominal pain, but it typically does not cause sepsis or decreased urine output. Measles can cause fever and a rash, but it typically does not cause sepsis or hemorrhage.   It is important to note that meningococcemia is a medical emergency that requires prompt recognition and treatment with antibiotics and supportive care, including fluid resuscitation and vasopressors if necessary. Close contacts of the patient should also receive prophylactic antibiotics to prevent further spread of the infection.  


28. A 35-year-old patient has been sick for the 3rd day. 5 days ago I returned from Africa. The disease began with fever up to 40°C, chills, severe headache, myalgia. In the axillary region, a lymph node enlarged to 3×6 cm is palpated, dense, sharply painful, inactive, without clear contours, the skin above it is hyperemic, tense. Tachycardia. What is the preliminary diagnosis? select one:

A. Lymphadenitis

b. Plague

C. Tularemia

D. anthrax

E. Sepsis


The correct answer is: Plague

Explanation

The patient’s symptoms, including fever, chills, severe headache, myalgia, and a painful, enlarged lymph node with hyperemic, tense skin, suggest a diagnosis of plague. Plague is a bacterial infection caused by Yersinia pestis that is typically transmitted through flea bites and can cause a range of symptoms, including fever, chills, headache, myalgia, and lymphadenopathy.

Plague is endemic in certain parts of Africa, particularly in rural areas, and can be fatal if left untreated.   Lymphadenitis, tularemia, anthrax, and sepsis are possible differential diagnoses but are less likely given the patient’s symptoms and history of recent travel to Africa. Lymphadenitis refers to inflammation of the lymph nodes and can be caused by a variety of infectious and non-infectious causes, but it typically does not cause systemic symptoms such as fever and chills.  

Tularemia is a bacterial infection caused by Francisella tularensis and can cause fever, chills, and lymphadenopathy, but it typically does not cause the rapid onset of symptoms seen in plague.   Anthrax is a bacterial infection caused by Bacillus anthracis and can cause a range of symptoms, including fever, chills, and lymphadenopathy, but it typically presents with different clinical features such as skin lesions and respiratory symptoms.  

Sepsis can be caused by a variety of bacterial and viral infections, but it typically does not cause the characteristic lymphadenopathy seen in plague.   It is important to note that plague is a highly infectious and potentially fatal disease that requires prompt diagnosis and treatment with antibiotics, such as streptomycin or gentamicin. Close contacts of the patient should also be monitored and treated prophylactically if necessary.


29. A 45-year-old patient who underwent a mastectomy with chemo-radiation therapy a year ago complains of shortness of breath at rest, an increase in body temperature up to 37.2°C. The general condition is severe, acrocyanosis. The right half of the chest almost does not participate in the act of breathing, with percussion – a dull sound below the third rib, auscultatory – a sharp weakening of respiratory sounds. A large amount of hemorrhagic exudate was obtained during pleural puncture on the right. What complication did the patient have? select one:

A. Carcinomatous pleurisy

b. Right lung abscess

C. Acute right sided pleuropneumonia

D. Pulmonary embolism

E. Acute pleural empyema


The correct answer is: Carcinomatous pleurisy

Explanation

The patient’s symptoms, including shortness of breath at rest, increased body temperature, acrocyanosis, dull percussion sound and weakened respiratory sounds on the right side of the chest, and hemorrhagic pleural effusion, suggest a diagnosis of carcinomatous pleurisy. Carcinomatous pleurisy is a common complication of cancer, particularly lung cancer and breast cancer, and is characterized by the spread of cancer cells to the pleura, the thin membrane that lines the lungs and chest cavity.

The presence of pleural effusion, especially hemorrhagic effusion, is a common feature of carcinomatous pleurisy.   Right lung abscess, acute right-sided pleuropneumonia, pulmonary embolism, and acute pleural empyema are possible differential diagnoses but are less likely given the patient’s symptoms and findings. Right lung abscess is a localized collection of pus within the lung tissue and typically presents with fever, cough, and chest pain, but it does not typically cause pleural effusion or acrocyanosis.

Acute right-sided pleuropneumonia is an inflammatory condition of the lung and pleura and can cause fever, chest pain, and cough, but it typically does not cause acrocyanosis or hemorrhagic pleural effusion. Pulmonary embolism is a blockage of the pulmonary artery by a blood clot and can cause shortness of breath and chest pain, but it typically does not cause pleural effusion or acrocyanosis.

Acute pleural empyema is a collection of pus within the pleural space and can cause fever, chest pain, and shortness of breath, but it typically does not cause acrocyanosis or hemorrhagic effusion.   It is important to note that carcinomatous pleurisy is a serious condition that can cause significant morbidity and mortality if left untreated. Treatment typically involves a combination of chemotherapy, radiation therapy, and supportive care, including drainage of the pleural effusion if necessary.  


30. The patient is 28 years old. He has been suffering from a mental disorder since the age of 22. The current state has changed dramatically, for 3 days the patient did not sleep, he was afraid to leave the house. He stated that “telepathy” is formed between him and other people, through which “other people’s thoughts” are transmitted to his head, and his own thoughts become known to everyone. He believes that under the influence of “telepathy” his thoughts and actions are controlled. What is the preliminary diagnosis? select one:

A. Acute reaction to stress

b. paranoid schizophrenia

C. manic episode

D. organic delirium

E. depressive episode


The correct answer is: Paranoid schizophrenia

Explanation

The patient’s symptoms, including the onset of a mental disorder at a young age, delusions of telepathy and control, and changes in behavior such as insomnia and fear of leaving the house, suggest a diagnosis of paranoid schizophrenia. Paranoid schizophrenia is a subtype of schizophrenia characterized by the presence of delusions, particularly delusions of persecution or grandeur, and auditory or visual hallucinations. Paranoid schizophrenia typically presents in young adulthood and can cause significant impairment in social and occupational functioning.  

Acute reaction to stress, manic episode, organic delirium, and depressive episode are possible differential diagnoses but are less likely given the patient’s symptoms and history. Acute reaction to stress, also known as acute stress disorder, is a short-term reaction to a traumatic event and typically presents with symptoms such as anxiety, dissociation, and re-experiencing of the traumatic event.

Manic episode is a part of bipolar disorder and is characterized by elevated or irritable mood, increased energy, decreased need for sleep, and grandiose or risky behavior. Organic delirium refers to a disturbance in consciousness and cognition caused by a medical condition, such as infection or drug toxicity.

Depressive episode is a part of major depressive disorder and is characterized by depressed mood, loss of interest or pleasure, and changes in appetite, sleep, and energy.   It is important to note that paranoid schizophrenia is a chronic and severe mental illness that requires long-term treatment with antipsychotic medication and psychosocial interventions, such as cognitive-behavioral therapy and family therapy. Early recognition and treatment of schizophrenia can improve outcomes and prevent complications such as suicide, substance abuse, and social isolation.  


31. A 45-year-old patient has been suffering from rheumatoid arthritis (RA) for 10 years and takes methotrexate daily. Which statement about pneumococcal vaccination (23-valent vaccine) in this case is consistent with the recommendations of the European Antirheumatic League (2010)? select one:

A. Vaccination not recommended

b. Vaccination recommended

C. Vaccination requires an increase in doses of basic drugs

D. Vaccination is contraindicated in patients taking methotrexate

E. Vaccination is contraindicated in the activity of the inflammatory process


The correct answer is: Vaccination recommended

Explanation

The statement consistent with the recommendations of the European Antirheumatic League (2010) regarding pneumococcal vaccination (23-valent vaccine) in a patient with rheumatoid arthritis (RA) taking methotrexate is that vaccination is recommended.  

Patients with RA have an increased risk of infections, including pneumococcal infections, due to the disease itself and the use of immunosuppressive medications such as methotrexate. The 23-valent pneumococcal vaccine is recommended for adults with underlying medical conditions, including RA, to reduce the risk of pneumococcal infections.  

There is no need to increase the doses of basic drugs in order to receive the pneumococcal vaccine. Methotrexate does not contraindicate pneumococcal vaccination, although caution should be taken in patients with severe immunosuppression and vaccination should be timed appropriately to minimize the risk of infection.  

In summary, patients with RA taking methotrexate should receive the 23-valent pneumococcal vaccine according to the recommendations of the European Antirheumatic League to reduce the risk of pneumococcal infections.


32. You have witnessed an accident. During the inspection of the scene, they saw a man, about 30 years old, who was hit by a car. Unconscious. There is profuse bleeding on the left side of the neck, bright red. How should the bleeding be stopped? select one:

A. Put a splint on your neck

b. Maximum neck extension

C. Move to a stable position

D. Finger pressure, Mikulich method

E. Put on a plaster cast


The correct answer is: Finger pressure, Mikulich method

Explanation

The bleeding from the left side of the neck should be stopped using finger pressure, specifically the Mikulicz method. This is a first aid technique used to control bleeding from a neck wound. To perform the Mikulicz method, the rescuer should apply firm pressure to the site of bleeding using the fingers, preferably the index and middle fingers, for 10-15 minutes until the bleeding stops or medical help arrives.

It’s important to note that the Mikulicz method should not be used in cases of arterial bleeding, as it can cause the formation of a false aneurysm.   Applying a splint to the neck, extending the neck, or putting on a plaster cast are not appropriate first aid measures in this situation and may even cause harm to the patient.

Moving the patient to a stable position is important to prevent further injury and to provide necessary assistance, but stopping the bleeding should be the first priority.   It’s important to call for emergency medical services (EMS) immediately after stopping the bleeding. The patient is likely to require further medical attention and may need to be transported to a hospital for evaluation and treatment.  


33. A 60-year-old woman complains of unbearable pain in the right hypochondrium. Sick for 2 years. Worse after a festive meal. Objectively: yellowness of the sclera. There are no symptoms of peritoneal irritation. Positive symptoms of Ortner, Gubergritsa-Skulsky. Urine diastasis – 320 units. What is the most likely diagnosis? select one:

A. Pancreas cancer b. Acute cholangitis

C. Acute cholecystitis

D. Chronic cholecystitis

E. Chronic pancreatitis (exacerbation)


The correct answer is: Chronic pancreatitis (exacerbation)

Explanation

The most likely diagnosis for this patient is chronic pancreatitis with exacerbation. Chronic pancreatitis is a long-term inflammation of the pancreas that leads to the destruction of pancreatic tissue and impairment of pancreatic function. It typically presents with severe abdominal pain, often in the right upper quadrant or epigastric region, and can be exacerbated by alcohol intake or large meals. The yellowing of the sclera (jaundice) suggests that there is a blockage in the biliary system, which can be a complication of chronic pancreatitis.  

The positive symptoms of Ortner (hoarseness due to recurrent laryngeal nerve palsy) and Gubergritsa-Skulsky (pain in the right hypochondrium upon palpation) are associated with chronic pancreatitis and support this diagnosis. The high urine diastase levels also suggest pancreatic inflammation.   Acute cholangitis and acute cholecystitis are possible differential diagnoses, but they are less likely given the chronic nature of the patient’s symptoms. Acute cholangitis is an acute inflammation of the bile ducts typically caused by a bacterial infection, and can cause fever, abdominal pain, and jaundice.

Acute cholecystitis is an acute inflammation of the gallbladder typically caused by gallstones, and can cause severe right upper quadrant pain, fever, and leukocytosis.   Chronic cholecystitis is also a possible differential diagnosis, but the yellowing of the sclera and high urine diastase levels suggest pancreatic involvement.

Chronic cholecystitis is a chronic inflammation of the gallbladder typically caused by gallstones, and can cause recurrent episodes of right upper quadrant pain, nausea, and vomiting.   It is important to note that chronic pancreatitis is a serious condition that requires ongoing management to prevent complications such as pancreatic insufficiency, malnutrition, and diabetes.

Treatment typically involves lifestyle modifications such as abstinence from alcohol and a low-fat diet, as well as medications to manage symptoms and prevent complications.  


34. A 44-year-old patient complains of compressive pain behind the sternum, shortness of breath, periodic dizziness. Examination revealed hypertrophic obstructive cardiomyopathy with diastolic variant of heart failure. What medicines should be preferred? select one:

A. β-blockers

b. ACE inhibitors

C. Diuretics

D. Nitrates

E. cardiac glycosides


The correct answer is: β-blockers

Explanation

The preferred medication for a patient with hypertrophic obstructive cardiomyopathy with diastolic variant of heart failure is β-blockers.   Hypertrophic obstructive cardiomyopathy (HOCM) is a genetic disorder that causes thickening of the heart muscle, particularly the ventricular septum, leading to obstruction of blood flow and impaired diastolic function.

β-blockers are the first-line therapy for HOCM as they reduce the heart rate, decrease myocardial contractility, and improve diastolic filling, which can alleviate symptoms such as chest pain, shortness of breath, and dizziness.   ACE inhibitors and diuretics may also be used in the treatment of heart failure, but they are less effective in cases of HOCM with diastolic dysfunction. ACE inhibitors can improve left ventricular function and reduce myocardial hypertrophy, but they can also cause hypotension and worsen symptoms of HOCM.

Diuretics can reduce fluid overload and improve symptoms of heart failure, but they can also cause hypotension and electrolyte imbalances.   Nitrates are not typically used in the treatment of HOCM as they can worsen the obstruction of blood flow and cause hypotension.   Cardiac glycosides, such as digoxin, may be used in the treatment of heart failure, but they are not typically used as first-line therapy for HOCM with diastolic dysfunction.  

It is important to note that the management of HOCM is individualized and may include other medications, such as calcium channel blockers or disopyramide, depending on the patient’s symptoms and response to treatment. Regular monitoring and follow-up with a cardiologist are also important to prevent complications such as arrhythmias and sudden cardiac death.  


35. During the winter influenza epidemic caused mainly by influenza A H1N1 (04/2009 California), a 30-year-old patient was hospitalized with clinical signs of influenza on the 2nd day of illness: high fever, dry cough, myalgia, headache, general weakness. What should be prescribed as etiotropic therapy? select one:

A. Interferon inducers

b. Neuraminidase inhibitors (oseltamevir)

C. Acyclovir

D. Antibiotics

E. Immunoglobulin


The correct answer is: Neuraminidase inhibitors (oseltamevir)

Explanation

The preferred etiotropic therapy for a patient with clinical signs of influenza caused mainly by influenza A H1N1 is neuraminidase inhibitors (oseltamivir).   Neuraminidase inhibitors are antiviral medications that inhibit the replication of influenza viruses, including the H1N1 strain. They are most effective when administered within the first 48 hours of symptom onset, which is why early diagnosis and prompt treatment are important.  

Interferon inducers, such as interferon alpha, may also be used in the treatment of influenza, but they are less effective than neuraminidase inhibitors and may cause significant side effects.   Acyclovir is an antiviral medication used to treat herpes virus infections, such as herpes simplex and varicella-zoster virus, but it is not effective against influenza viruses.  

Antibiotics are not effective against viral infections such as influenza and should not be prescribed unless there is evidence of bacterial co-infection or complications such as pneumonia.   Immunoglobulin may be used in the treatment of severe influenza or in high-risk patients who are unable to receive the influenza vaccine, but it is not typically used as first-line therapy for uncomplicated influenza.  

In summary, a patient with clinical signs of influenza caused mainly by influenza A H1N1 should be treated with neuraminidase inhibitors such as oseltamivir, especially if administered within the first 48 hours of symptom onset.  


36.A 4-month-old child with meningococcemia at the time of admission: acrocyanosis, cold extremities, tachypnea, thready pulse, blood pressure – 30/0 mm Hg, anuria, stupor. What is the clinical syndrome in the child? select one:

A. exsicosis

b. Infectious-toxic shock

C. encephalic syndrome

D. Neurotoxicosis

E. Acute renal failure


The correct answer is: Toxic Infectious Shock

Explanation

The clinical syndrome in the child is toxic infectious shock.   Meningococcemia is a severe bacterial infection caused by Neisseria meningitidis that can cause sepsis, meningitis, or both. Toxic infectious shock is a life-threatening condition that can develop as a complication of meningococcemia or other severe bacterial infections.

It is characterized by hypotension, tachycardia, fever, and multiple organ dysfunction, including acute renal failure, which can result in anuria. The acrocyanosis, cold extremities, thready pulse, and stupor in the child are all consistent with the clinical presentation of toxic infectious shock.   Exsicosis refers to dehydration and is not a specific clinical syndrome.  

Encephalic syndrome and neurotoxicosis both refer to neurological disorders, which may be present in meningococcal meningitis but are not specific to toxic infectious shock.  

In summary, a 4-month-old child with meningococcemia presenting with acrocyanosis, cold extremities, tachypnea, thready pulse, hypotension, anuria, and stupor is most likely experiencing toxic infectious shock, which is a serious and potentially life-threatening condition that requires immediate medical attention and aggressive management.  


37. A 20-year-old patient complains of a feeling of lack of air, prolonged aching pain in the region of the heart, and irritability. Objectively: the general condition is satisfactory, pulse lability, blood pressure – 130/60 mm Hg. ECG – violation of repolarization processes. The patient was diagnosed with neurocirculatory dystonia of the cardiac type. Specify the conditions under which the patient should receive treatment: select one:

A. Inpatient treatment in the therapeutic department

b. Ambulatory treatment

C. Inpatient treatment in the cardiology department

D. Inpatient treatment in the cardiac surgery department

E. Inpatient treatment in a psychiatric ward


The correct answer is: Outpatient care

Explanation

The patient with neurocirculatory dystonia of the cardiac type can generally be managed with ambulatory treatment.   Neurocirculatory dystonia (NCD) is a functional disorder of the autonomic nervous system that can cause a range of symptoms, including palpitations, chest pain, shortness of breath, and anxiety. NCD is a benign condition and does not cause significant damage to the heart or other organs.

Treatment for NCD is typically focused on managing symptoms and may include lifestyle modifications (such as stress reduction and regular exercise), medications to control symptoms (such as beta-blockers or anxiolytics), and psychological support.   In the case of a 20-year-old patient with NCD of the cardiac type, the symptoms described (feeling of lack of air, prolonged aching pain in the region of the heart, and irritability) suggest a mild to moderate form of the condition. The patient’s general condition is satisfactory, and the blood pressure is within the normal range.

The ECG findings are consistent with NCD.   Therefore, this patient can be managed with ambulatory treatment, which may include regular follow-up with a primary care physician or cardiologist, lifestyle modifications, and medications as needed. Inpatient treatment is typically reserved for patients with severe symptoms, complications, or comorbidities that require more intensive monitoring or intervention.  

In summary, a patient with neurocirculatory dystonia of the cardiac type can generally be managed with ambulatory treatment, especially if the symptoms are mild to moderate and there are no significant complications or comorbidities.  


38. A 45-year-old patient, who underwent a heart operation a week ago, feels worse: he is worried about shortness of breath at rest, pain behind the sternum with irradiation to the neck, severe weakness. Objectively: hectic body temperature. The boundaries of the heart are expanded, the apex beat is weakened. Auscultatory – pericardial friction rub. Most likely diagnosis: select one:

A. Acute myogenic dilatation of the heart

b. Acute pericarditis

C. Acute cardiac aneurysm

D. Pulmonary embolism

E. myocardial infarction


The correct answer is: Acute pericarditis

Explanation

The most likely diagnosis for the patient in this scenario is acute pericarditis.   Acute pericarditis is an inflammation of the pericardium, which is the sac that surrounds the heart. It can occur as a complication of heart surgery and is characterized by sudden onset of chest pain, often with radiation to the neck and shoulders, shortness of breath, and fever. Pericardial friction rub may be heard on auscultation, and the boundaries of the heart may be expanded.  

Acute myogenic dilatation of the heart, acute cardiac aneurysm, and pulmonary embolism are less likely diagnoses in this case, as they typically present with different symptoms and physical examination findings. Acute myogenic dilatation of the heart refers to sudden dilation of the heart chambers, which can cause heart failure and shock, but it is a rare complication of heart surgery. Acute cardiac aneurysm refers to a bulge or protrusion in the heart wall, which can cause chest pain and heart failure, but it is also a rare complication of heart surgery.

Pulmonary embolism, which is a blockage of the pulmonary artery by a blood clot, can cause chest pain and shortness of breath, but it typically does not cause pericardial friction rub or fever.   Myocardial infarction is a possible differential diagnosis, as it can cause chest pain, shortness of breath, and weakness, but it typically presents with ECG changes (such as ST-segment elevation or depression) and elevated cardiac enzymes, which were not mentioned in the scenario.  

In summary, a patient who underwent heart surgery and presents with sudden onset of chest pain, shortness of breath, fever, pericardial friction rub, and expanded heart boundaries is most likely experiencing acute pericarditis, which is a common complication of heart surgery. Prompt diagnosis and treatment with nonsteroidal anti-inflammatory drugs (NSAIDs) and colchicine are important to prevent complications such as cardiac tamponade and constrictive pericarditis.


39. A 5-year-old child has been sick for 2 weeks. First came coughing fits, then reprises. During coughing, the patient’s face turns red, the veins of the neck swell. Coughing fits end in vomiting. On the radiograph: increased bronchial pattern. Blood test: leukocytes – $16cdot10^9$/l, lymph – 72%, erythrocyte sedimentation rate – 4 mm/hour. What is the most likely diagnosis? select one:

A. adenovirus infection

b. Whooping cough

C. Pneumonia

D. foreign body

E. Obstructive bronchitis


The correct answer is: Whooping Cough

Explanation

The most likely diagnosis for the 5-year-old child in this scenario is whooping cough.   Whooping cough, also known as pertussis, is a highly contagious bacterial infection caused by Bordetella pertussis. It is characterized by coughing fits followed by a characteristic “whooping” sound during inspiration, which can sometimes end in vomiting. The coughing fits can cause the face to turn red and the neck veins to swell. The leukocytosis with lymphocytosis and low erythrocyte sedimentation rate are common laboratory findings in pertussis.  

Adenovirus infection can cause respiratory symptoms, but it typically does not cause the characteristic coughing fits and whooping sound seen in pertussis.   Pneumonia can also cause coughing and respiratory symptoms, but it typically presents with fever, chest pain, and abnormal lung sounds on physical examination.

The increased bronchial pattern on radiograph is non-specific and can be seen in both pneumonia and obstructive bronchitis.   Foreign body aspiration can cause respiratory symptoms, but it typically presents with sudden onset of choking and wheezing, rather than a prolonged cough and whooping sound.  

Obstructive bronchitis can cause coughing and respiratory symptoms, but it typically presents with fever, wheezing, and abnormal lung sounds on physical examination. The leukocytosis with lymphocytosis and low erythrocyte sedimentation rate are not typical findings in obstructive bronchitis.  

In summary, a 5-year-old child with coughing fits followed by a characteristic whooping sound, as well as red face, swollen neck veins, and vomiting, is most likely experiencing whooping cough, which is a highly contagious bacterial infection caused by Bordetella pertussis. Early diagnosis and treatment with antibiotics are important to prevent complications and transmission to others.  


40. What modern organizational method makes it possible to provide patients with timely access to medical care of adequate quality, to receive medical services for counseling, diagnosis and treatment in remote settlements, especially in cases where distance and time are critical factors in the provision of medical care? select one:

A. Mobile teams of specialists

b. Telemedicine

C. air ambulance

D. mobile connection

E. Ambulance


The correct answer is: Telemedicine

Explanation

The modern organizational method that makes it possible to provide patients with timely access to medical care of adequate quality, to receive medical services for counseling, diagnosis, and treatment in remote settlements, especially in cases where distance and time are critical factors in the provision of medical care, is telemedicine.  

Telemedicine is the use of telecommunication and information technologies to provide health care services remotely. It allows healthcare professionals to diagnose, consult, and treat patients in remote or underserved areas, using audio, video, and other communication technologies.

Telemedicine can provide access to medical care for patients who may otherwise have limited or no access to healthcare services due to geographic, economic, or other barriers.   Mobile teams of specialists, air ambulance, mobile connection, and ambulance are also important components of modern healthcare systems, but they may not be as effective in providing timely access to medical care in remote or underserved areas.

Mobile teams of specialists and air ambulance services require physical transportation of healthcare professionals and equipment, which can be limited by weather conditions, terrain, or other factors. Mobile connection and ambulance services can facilitate communication and transportation, but they may not provide the level of expertise and specialized care that telemedicine can offer.  

In summary, telemedicine is a modern organizational method that can provide patients with timely access to medical care of adequate quality, especially in remote or underserved areas where distance and time are critical factors in the provision of medical care. Telemedicine can improve patient outcomes, reduce healthcare costs, and increase access to medical care for underserved populations.  


41.In a patient with suspected pheochromocytoma in the period between attacks of blood pressure in the normal range, there is a tendency to tachycardia. There is no pathology in the urine. It was decided to conduct a provocative test with histamine. What drug should be taken to provide emergency assistance in case of a positive test result? select one:

A. Prednisolone

b. Phentolamine

C. Nifedipine

D. Pipolfen

E. Mezaton


The correct answer is: Phentolamine

Explanation

The drug that should be taken to provide emergency assistance in case of a positive provocative test with histamine in a patient with suspected pheochromocytoma is phentolamine.   Pheochromocytoma is a rare tumor that arises from chromaffin cells in the adrenal medulla or sympathetic ganglia and secretes excessive amounts of catecholamines, such as adrenaline and noradrenaline.

The excess catecholamines can cause episodic or sustained hypertension, as well as tachycardia, sweating, headache, and other symptoms. Provocative tests, such as the histamine provocation test, can be used to trigger a release of catecholamines and confirm the diagnosis of pheochromocytoma.  

Phentolamine is an alpha-adrenergic blocker that can be used to treat hypertensive crises caused by pheochromocytoma. It works by blocking the action of catecholamines on alpha-adrenergic receptors, thereby reducing vasoconstriction and lowering blood pressure. In the case of a positive histamine provocation test, phentolamine can be administered to rapidly reduce blood pressure and prevent complications such as stroke, myocardial infarction, or pulmonary edema.  

Prednisolone is a glucocorticoid that can be used to treat inflammation and autoimmune disorders, but it is not effective in treating hypertensive crises caused by pheochromocytoma.   Nifedipine is a calcium channel blocker that can be used to treat hypertension, but it is not recommended for the treatment of hypertensive crises caused by pheochromocytoma, as it can cause reflex tachycardia and worsen symptoms.   Pipolfen is an antihistamine that can be used to treat allergies and motion sickness, but it is not effective in treating hypertensive crises caused by pheochromocytoma.  

Mezaton, also known as phenylephrine, is a sympathomimetic drug that can be used to treat hypotension, but it is contraindicated in the treatment of hypertensive crises caused by pheochromocytoma, as it can exacerbate symptoms by stimulating the release of catecholamines.   In summary, phentolamine is the drug of choice for emergency management of hypertensive crises caused by pheochromocytoma, especially in the context of a positive histamine provocation test.


42. During a routine examination, an 8-year-old girl with type I diabetes mellitus was found to have a swelling on the anterior surface of her right thigh measuring 3 cm in diameter, dense, painless on palpation. Skin over the formation of normal color and temperature. The localization of the swelling coincides with the place where insulin is most often injected. What is the most likely cause of such a clinical picture? select one:

A. With the development of atrophic lipodystrophy

b. With an allergic reaction

C. With the formation of post-injection abscess

D. With the formation of post-injection infiltrate

E. With the development of hypertrophic lipodystrophy


The correct answer is: With the development of hypertrophic lipodystrophy

Explanation

The most likely cause of the swelling on the anterior surface of the 8-year-old girl’s right thigh, which coincides with the place where insulin is most often injected, is the development of hypertrophic lipodystrophy.   Hypertrophic lipodystrophy, also known as lipoatrophy or insulin-induced lipodystrophy, is a relatively common complication of insulin injections in patients with diabetes mellitus.

It is characterized by the development of a painless, firm, and often disfiguring lump at the injection site due to the buildup of fibrous tissue and loss of subcutaneous fat. The skin over the lump is typically normal in color and temperature.   Atrophic lipodystrophy, on the other hand, is characterized by a loss of fat tissue at the injection site, which can cause the skin to become thin, wrinkled, and discolored.

Post-injection abscess and post-injection infiltrate are less likely diagnoses, as they typically present with redness, warmth, tenderness, and other signs of inflammation at the injection site. An allergic reaction can also cause inflammation and redness at the injection site, but it is typically associated with other symptoms such as itching, hives, and difficulty breathing.  

In summary, the most likely cause of the painless swelling on the anterior surface of the 8-year-old girl’s right thigh, which coincides with the place where insulin is most often injected, is hypertrophic lipodystrophy, a common complication of insulin injections in patients with diabetes mellitus. Regular rotation of injection sites and careful monitoring of injection technique can help to prevent the development of lipodystrophy and other injection site complications.  


43. In a premature infant with respiratory failure syndrome and malnutrition, the attending pediatrician detected a continuous systolicdiastolic heart murmur, most pronounced in the second intercostal space on the left. On the radiograph, an increase in the pulmonary vascular pattern, an increase in the left ventricle and an expansion of the shadow of the pulmonary arteries are found. What malformation is most likely in a child? select one:

A. aortic stenosis

b. Tetralogy of Fallot

C. Open ductus arteriosus

D. Transposition of the great vessels

E. Ventricular septal defect


The correct answer is: Patent ductus arteriosus

Explanation

The most likely malformation in the premature infant with respiratory failure syndrome and malnutrition who presents with a continuous systolic-diastolic heart murmur, most pronounced in the second intercostal space on the left, an increase in the pulmonary vascular pattern, an increase in the left ventricle, and an expansion of the shadow of the pulmonary arteries on the radiograph, is patent ductus arteriosus.  

Patent ductus arteriosus (PDA) is a common congenital heart defect that occurs when the ductus arteriosus, a blood vessel that connects the pulmonary artery to the aorta in a fetus, fails to close after birth. This results in the continuous flow of oxygenated blood from the aorta into the pulmonary artery, causing an increase in pulmonary blood flow and left ventricular volume overload.

The continuous systolic-diastolic heart murmur heard on auscultation is due to the turbulent flow of blood through the PDA.   Aortic stenosis, Tetralogy of Fallot, Transposition of the great vessels, and Ventricular septal defect are all other types of congenital heart defects, but they are less likely to present with the characteristic findings seen in PDA, such as a continuous systolic-diastolic heart murmur, increased pulmonary vascular pattern, and left ventricular volume overload.  

In summary, the most likely malformation in the premature infant with respiratory failure syndrome and malnutrition who presents with a continuous systolic-diastolic heart murmur, most pronounced in the second intercostal space on the left, an increase in the pulmonary vascular pattern, an increase in the left ventricle, and an expansion of the shadow of the pulmonary arteries on the radiograph, is patent ductus arteriosus. Early recognition and treatment of PDA are important to prevent complications such as pulmonary hypertension, congestive heart failure, and endarteritis.  


44. A 39-year-old patient, who suffers from chronic adrenal insufficiency and receives replacement therapy with glucocorticoids (hydrocortisone – 15 mg/day), is scheduled for surgery due to calculous cholecystitis. What treatment is prescribed on the day of surgery to prevent acute adrenal insufficiency? select one:

A. Antibiotic added to treatment

b. Mineralocorticoid added to treatment

C. Infusion of large volumes of fluid

D. On the day of surgery, the drug is canceled

E. The dose of the drug is increased by 2 times


The correct answer is: The dose of the drug is increased by 2 times

Explanation

In a patient with chronic adrenal insufficiency who is receiving replacement therapy with glucocorticoids and is scheduled for surgery, the appropriate treatment on the day of surgery to prevent acute adrenal insufficiency is to increase the dose of the drug by 2 times.   Chronic adrenal insufficiency, also known as Addison’s disease, is a condition in which the adrenal glands do not produce enough cortisol and sometimes aldosterone.

Glucocorticoid replacement therapy with hydrocortisone is the mainstay of treatment for this condition. Patients with chronic adrenal insufficiency are at risk for acute adrenal crisis during times of stress, such as surgery, trauma, or illness.   To prevent acute adrenal crisis during surgery, it is recommended to increase the dose of glucocorticoids on the day of surgery. The dose should be increased by 2-3 times the usual dose, depending on the severity of the surgery and the patient’s individual needs.

This increased dose should be continued for 24-48 hours after surgery, depending on the patient’s postoperative recovery.   Antibiotics, mineralocorticoids, and large volumes of fluid are not generally indicated for the prevention of acute adrenal crisis in patients with chronic adrenal insufficiency undergoing surgery.

Canceling the drug on the day of surgery is not appropriate and can lead to acute adrenal crisis.   In summary, the appropriate treatment on the day of surgery to prevent acute adrenal insufficiency in a patient with chronic adrenal insufficiency who is receiving replacement therapy with glucocorticoids is to increase the dose of the drug by 2 times. This increased dose should be continued for 24-48 hours after surgery, depending on the patient’s postoperative recovery.  


45. Assessing the state of health of drivers and road traffic inspectors, doctors found the presence of carboxyhemoglobin in the blood of the subjects, a decrease in their reflex reaction, and a violation of the activity of a number of enzymes. The detected health disorders in people of these professional categories are most likely associated with exposure to: select one:

A. Neuro-emotional stress

b. carbon monoxide

C. aromatic carbohydrates

D. Sulfur dioxide

E. nitrogen oxides


The correct answer is: Carbon monoxide

Explanation

The health disorders detected in drivers and road traffic inspectors, such as the presence of carboxyhemoglobin in the blood, a decrease in their reflex reaction, and a violation of the activity of some enzymes, are most likely associated with exposure to carbon monoxide.  

Carbon monoxide (CO) is a toxic gas that is produced by the incomplete combustion of carbon-containing fuels, such as gasoline, diesel, and natural gas. It can accumulate in enclosed spaces, such as garages, tunnels, and poorly ventilated workplaces, and can be inhaled by people, leading to the formation of carboxyhemoglobin in the blood. Carboxyhemoglobin is a stable complex of carbon monoxide and hemoglobin that impairs the ability of red blood cells to transport oxygen to tissues, leading to tissue hypoxia and a wide range of symptoms, including headache, dizziness, nausea, confusion, and impaired reflexes.  

Drivers and road traffic inspectors, who spend significant amounts of time in or around motor vehicles and traffic, are at increased risk of exposure to carbon monoxide. The symptoms of carbon monoxide poisoning can impair their ability to operate vehicles safely and react to unexpected situations on the road, leading to an increased risk of accidents and injuries.  

Neuro-emotional stress, aromatic carbohydrates, sulfur dioxide, and nitrogen oxides are not known to cause the specific health disorders described in the question. While exposure to these agents may cause other types of health problems, such as respiratory and cardiovascular diseases, they are not likely to cause the specific symptoms of carboxyhemoglobinemia and impaired reflexes seen in this case.  

In summary, the health disorders detected in drivers and road traffic inspectors, such as the presence of carboxyhemoglobin in the blood, a decrease in their reflex reaction, and a violation of the activity of some enzymes, are most likely associated with exposure to carbon monoxide, a toxic gas produced by the incomplete combustion of carbon-containing fuels.


46. A 63-year-old woman suddenly had an asthma attack at night. For about 15 years she has been suffering from hypertension, 2 years ago she suffered a myocardial infarction. Objectively: position in bed – orthopnea, pale skin, the patient is covered with cold sweat, acrocyanosis. Pulse – 104/min. Arterial pressure – 210/130 mm Hg, respiratory rate – 38/min. Percussion sound is pulmonary, dull in the lower sections, single dry rales are heard throughout, in the lower sections – muffled, finely bubbling. What complication most likely developed in the patient? select one:

A. Paroxysmal tachycardia

b. Acute left ventricular failure

C. Asthma attack

D. Pulmonary embolism

E. Acute right heart failure


The correct answer is: Acute left ventricular failure

Explanation

In the given scenario, the most likely complication developed in the patient is acute left ventricular failure.   The patient is a 63-year-old woman with a history of hypertension and myocardial infarction who suddenly developed an asthma attack at night. On examination, the patient is in the orthopneic position, has pale skin, cold sweat, and acrocyanosis.

The pulse rate is elevated, blood pressure is significantly elevated, and respiratory rate is increased. Auscultation of the lungs reveals dull percussion sounds in the lower sections, single dry rales throughout, and muffled, finely bubbling rales in the lower sections. These findings are consistent with the development of acute left ventricular failure.  

Acute left ventricular failure, also known as pulmonary edema, is a condition characterized by the accumulation of fluid in the lungs due to impaired left ventricular function. The patient’s symptoms, including orthopnea, tachycardia, and elevated blood pressure, are consistent with this diagnosis. The presence of crackles and dullness on lung auscultation further supports the diagnosis of acute left ventricular failure.  

Paroxysmal tachycardia is a type of arrhythmia that is characterized by episodes of rapid heart rate that start and stop suddenly. While the patient in this scenario has an elevated pulse rate, it is likely due to the acute left ventricular failure rather than paroxysmal tachycardia.   Asthma attack and pulmonary embolism can cause respiratory symptoms, but they are less likely to cause the cardiovascular symptoms and signs, such as elevated blood pressure, tachycardia, and acrocyanosis, seen in this scenario.

Acute right heart failure can also cause similar symptoms, but in this case, the patient’s lung auscultation findings are more consistent with left ventricular failure.   In summary, the most likely complication developed in the 63-year-old woman who suddenly had an asthma attack at night is acute left ventricular failure, a condition characterized by impaired left ventricular function leading to the accumulation of fluid in the lungs. Early recognition and treatment of acute left ventricular failure are important to prevent complications and improve outcomes.  


47. An 18-year-old patient consulted a doctor complaining of a skin rash. Sick for 5 years. The disease first appeared after a car accident. Objectively: papular rashes covered with silvery scales, nails with a thimble symptom, joint damage. What is the most likely diagnosis? select one:

A. Onychomycosis

b. Rheumatism

C. lupus erythematosus

D. Felon E. Psoriasis


The correct answer is: Psoriasis

Explanation

The most likely diagnosis in the 18-year-old patient with a skin rash, nail changes, and joint involvement is psoriasis.   Psoriasis is a chronic inflammatory skin disease characterized by well-defined, erythematous papules and plaques covered with silvery scales. It commonly affects the scalp, elbows, knees, and lower back. The thimble sign, which refers to the presence of small depressions on the surface of the nails, is a characteristic nail change seen in psoriasis. Joint involvement, known as psoriatic arthritis, is also common in psoriasis.  

The onset of psoriasis can be triggered by environmental factors, such as trauma, infection, and stress. In this case, the patient reports a history of a car accident preceding the onset of the disease, which could have triggered the development of psoriasis.   Onychomycosis is a fungal infection of the nails that can cause thickening and discoloration of the nails, but it is less likely to cause the thimble sign seen in this case.  

Rheumatism is a term used to describe various musculoskeletal disorders that affect the joints, bones, and muscles. While psoriatic arthritis can be considered a type of rheumatism, the presence of skin and nail changes makes psoriasis a more likely diagnosis in this case.   Lupus erythematosus is an autoimmune disease that can affect the skin, joints, and other organs.

While it can cause skin rashes, the presence of silvery scales and the thimble sign on the nails are not typical findings in lupus erythematosus.   Felon is an infection of the fingertip that can cause pain, swelling, and redness. It is not likely to cause skin rashes, nail changes, or joint involvement.  

In summary, the most likely diagnosis in the 18-year-old patient with a skin rash, nail changes, and joint involvement is psoriasis, a chronic inflammatory skin disease characterized by erythematous papules and plaques covered with silvery scales, thimble sign on the nails, and joint involvement. Treatment options for psoriasis include topical medications, phototherapy, and systemic medications.


48. A 66-year-old patient with deep vein thrombophlebitis of the extremities has increased shortness of breath after significant physical exertion, severe pain in the right half of the chest, pronounced palpitations. Severe condition, cyanosis of the face, swelling of the jugular veins. Blood pressure – 60/40 mm Hg. Which of the following methods is the most appropriate in this case? select one:

A. echocardiography

b. Selective angiopulmonography

C. chest MRI

D. X-ray of the lungs

E. Fibrobronchoscopy


The correct answer is: Selective angiopulmonography

Explanation

In the given scenario, the most appropriate method for diagnosis in the 66-year-old patient with deep vein thrombophlebitis of the extremities, shortness of breath, chest pain, palpitations, cyanosis, jugular vein distension, and low blood pressure is selective angiopulmonography.   The patient’s symptoms and signs are suggestive of a pulmonary embolism, a potentially life-threatening condition in which a blood clot travels to the lungs and obstructs blood flow.

The symptoms of pulmonary embolism include sudden onset of shortness of breath, chest pain, palpitations, and cyanosis. Jugular vein distension and low blood pressure are signs of hemodynamic instability and shock.   Selective angiopulmonography is an invasive diagnostic procedure that involves injecting contrast dye into the pulmonary arteries and taking X-rays to visualize the blood vessels in the lungs.

This test is considered the gold standard for the diagnosis of pulmonary embolism. It can identify the location and extent of the blood clot and help guide treatment.  

Echocardiography, chest MRI, and X-ray of the lungs are non-invasive diagnostic tests that can provide information about the heart and lungs, but they are less sensitive and specific than selective angiopulmonography for the diagnosis of pulmonary embolism.   Fibrobronchoscopy is a diagnostic procedure that involves inserting a flexible tube with a camera and light into the airways to visualize the lungs and collect samples for testing.

While it can help diagnose some lung conditions, it is not useful for the diagnosis of pulmonary embolism.   In summary, the most appropriate method for diagnosis in the 66-year-old patient with deep vein thrombophlebitis of the extremities, shortness of breath, chest pain, palpitations, cyanosis, jugular vein distension, and low blood pressure is selective angiopulmonography, an invasive diagnostic procedure that can provide definitive diagnosis of pulmonary embolism and guide treatment.  


49. The patient gradually lost consciousness. The skin is pale, dry. Ammonia smell from the mouth. Deep noisy breathing. Heart – weakened tones, pericardial friction noise, blood pressure – 180/130 mm Hg. In the blood: Hb – 80 g/l, leukocytes – $12cdot10^9 $/l, blood glucose – 6.4 mmol/l, urea – 50 mmol/l, creatinine – 1200 µmol/l, blood osmolarity – 350 mosm / l. Does not excrete urine. What is the most likely diagnosis? select one:

A. Hyperglycemic coma

b. Hyperosmolar coma

C. uremic coma

D. Acute cerebrovascular accident

E. Acute renal failure


The correct answer is: Uremic Coma

Explanation

The most likely diagnosis in the given case is uremic coma.   The patient has gradually lost consciousness with pale, dry skin, and deep, noisy breathing. There is an ammonia smell from the mouth, weakened heart tones, pericardial friction noise, and high blood pressure. Laboratory findings show low hemoglobin, high leukocytes, high blood glucose, high urea, high creatinine, and normal blood osmolarity.

The patient is not excreting urine.   These clinical findings and laboratory values are consistent with acute kidney injury (AKI) leading to uremic coma. AKI is a sudden decrease in kidney function resulting in the accumulation of metabolic waste products such as urea and creatinine in the blood. Uremic coma is characterized by mental confusion, seizures, and eventually coma, and is a serious complication of AKI.  

Hyperglycemic and hyperosmolar comas are complications of uncontrolled diabetes mellitus, but the laboratory findings in this case do not support a diagnosis of either of these conditions.   Acute cerebrovascular accident (stroke) can cause loss of consciousness, but the clinical and laboratory findings in this case are not consistent with this diagnosis.  

Acute renal failure is a broad term that refers to a sudden decrease in kidney function, which can lead to AKI and uremic coma. Therefore, while acute renal failure is a possible diagnosis in this case, uremic coma is a more specific and likely diagnosis based on the given clinical and laboratory findings.  

In summary, the most likely diagnosis in the given case of a patient with gradually decreasing consciousness, pale dry skin, deep noisy breathing, and laboratory findings consistent with acute kidney injury is uremic coma. Early recognition and treatment of AKI and its complications are important to improve outcomes.  


50.A 14-year-old girl is being examined by a pediatrician. Objectively found: a girl of high stature, asthenic build, stretch marks on the skin of the abdominal cavity, blue sclera. Scoliotic posture, chest deformity were diagnosed. Hypermobility of the joints, long fingers and hands are noted. On ultrasound of the heart, mitral valve prolapse is visualized. Which of the following is the most likely reason for her high stature?

a. Ehlers-Danlos syndrome

b. Marfan syndrome

c. Noonan syndrome

d. Williams syndrome

e. Klinefelter syndrome


The correct answer is: Marfan Syndrome

Explanation

The correct answer is b. Marfan syndrome.   Marfan syndrome is a genetic disorder that affects the connective tissue in the body, and it is characterized by tall stature, asthenic build, long fingers and hands, chest deformity, and scoliotic posture. Other common features of Marfan syndrome include hypermobility of the joints, stretch marks on the skin of the abdominal cavity, and blue sclera.  

Mitral valve prolapse is also a common finding in individuals with Marfan syndrome, which was visualized on the girl’s ultrasound of the heart.   Ehlers-Danlos syndrome is another genetic connective tissue disorder, which is characterized by hypermobility of the joints, skin hyperextensibility, and tissue fragility. However, it is not typically associated with tall stature or chest deformity.  

Noonan syndrome is a genetic disorder that affects multiple parts of the body, and it is characterized by short stature, webbed neck, and heart defects. This disorder is not associated with tall stature.   Williams syndrome is a genetic disorder that affects multiple parts of the body, and it is characterized by distinctive facial features, cardiovascular problems, and intellectual disability.

This disorder is not associated with tall stature.   Klinefelter syndrome is a genetic disorder that affects males, and it is characterized by an extra X chromosome, which leads to infertility, tall stature, and other physical and developmental abnormalities. However, Klinefelter syndrome is not typically associated with chest deformity or scoliotic posture.  

Join the conversation
0% Complete